mrudula_2005
Thanks Received: 21
Forum Guests
 
Posts: 136
Joined: July 29th, 2010
 
 
trophy
First Responder
 

PT 46, S3, Q3 - A recent study involved

by mrudula_2005 Sun Aug 22, 2010 2:53 pm

this is a technical/precision question:

Shouldn't D be worded to say "what percentage (instead of 'how many') of the rats in the colony studied had abnormally high blood pressure before the study began?" ? because we don't know how many rats are in the colony in the first place so how would an absolute number response help us? without knowing that there is X number of rats in the colony, how would it help us to know that there are 57 or 577 rats in the colony that already had abnormally high blood pressure?

thanks!
User avatar
 
ManhattanPrepLSAT1
Thanks Received: 1909
Atticus Finch
Atticus Finch
 
Posts: 2851
Joined: October 07th, 2009
 
 
 

Re: PT 46, S3, Q3 - A recent study involved

by ManhattanPrepLSAT1 Tue Aug 24, 2010 3:19 am

I see your point, but I think the issue you see isn't really there. This is a study of a colony of rats. I think it's fair to say that we know how many of the rats are in the study, and thus the colony.

Keep that sharp eye peeled for later in the section. Your pulling out too critical of an eye too early in the section! Good luck.
 
dtangie23
Thanks Received: 17
Jackie Chiles
Jackie Chiles
 
Posts: 27
Joined: September 29th, 2010
 
 
trophy
Most Thanked
 

Re: PT 46, S3, Q3 - A recent study involved

by dtangie23 Sun Jan 09, 2011 11:24 pm

What about answer choice (A)? Suppose the amount of salt in the high salt diet is roughly the same as that contained in a rat's normal diet. That would totally undermine the linkage making (A) a very relevant question.

Is (A) incorrect because it talks about "a rat's normal diet" rather than "a colony rat's normal diet?"
User avatar
 
ManhattanPrepLSAT1
Thanks Received: 1909
Atticus Finch
Atticus Finch
 
Posts: 2851
Joined: October 07th, 2009
 
This post thanked 1 time.
 
 

Re: PT 46, S3, Q3 - A recent study involved

by ManhattanPrepLSAT1 Mon Jan 24, 2011 5:31 am

I think the issue here is that the question is asking for which answer choice would be the most useful in evaluating the argument. Generally, you're looking for a question that can be answered in several ways and that the answer in one direction would support the conclusion, whereas an answer in the other direction would undermine the conclusion.

The biggest problem with this argument is that there is no increased correlation established high salt diets and high blood pressure in rats. Suppose 90% of rats with a normal diet have high blood pressure, then high salt would actually have a negative correlation with high blood pressure.

The question we should be asking is, "is the blood pressure distribution that was observed elevated, unchanged, or reduced from the normal situation?"

Answer choice (A) is incorrect, because regardless of how much more salt a high-salt diet possesses, it still possesses enough to be considered high-salt. The effects of a high-salt diet are much more important to determine here than what constitutes a high-salt diet.
 
jimmy902o
Thanks Received: 4
Elle Woods
Elle Woods
 
Posts: 90
Joined: August 06th, 2011
 
 
 

Re: Q3 - A recent study involved

by jimmy902o Tue Oct 23, 2012 8:13 pm

Matt, can you please go into more detail as to why A is wrong?

Also if you could explain the difference between this and PT 59/Section 2/Question 22? It seems to me that if we apply the technique used in this question (affirming pretest concerns) this would lead us to choosing C (in test 59) which is the wrong answer. Thoughts?
 
jimmy902o
Thanks Received: 4
Elle Woods
Elle Woods
 
Posts: 90
Joined: August 06th, 2011
 
 
 

Re: Q3 - A recent study involved

by jimmy902o Tue Oct 23, 2012 8:34 pm

on second thought i may have answered my own question... is it because the correlation in pt 59 is not in question?
 
etwcho
Thanks Received: 12
Forum Guests
 
Posts: 27
Joined: February 24th, 2013
 
 
 

Re: Q3 - A recent study involved

by etwcho Fri May 10, 2013 4:04 am

I'm terrible at this type, so I'm going to attempt to dissect this question on here to help myself and possibly others. Please let me know if something seems off.

The core of the argument hinges on a relationship in which, the author concludes that high-salts diets are linked to high blood pressure in rats, based on evidence that after several months of feeding a rat colony high-salt diets, 25%, 70%, and 5% of the colony had normal, high and extremely high blood pressure, respectively.
Because the stem asks us to find a question that most helps to evaluate the conclusion, our task is to find a question that could prove/disprove the connection between the high-salt diet and high blood pressure.

D) While I was reading the stimulus, I was very skeptical of the whole experiment because the evidence didn't explain the change. All it does is giving the distribution of different levels of blood pressure post-experiment and nothing is given as to what the case was before. What if the distribution of different levels of blood pressure were same previous to the experiment? That would mean nothing changed and hence there would be no reason to conclude a relationship.
For example, if I met Paul, a friend who I haven’t seen for 6 years, and said that he changed, a 3rd person wouldn’t have a clue as to whether he actually changed or not until getting the answer to the question "Well, what was he like 6 years ago?"

A) Incorrect, because the amount of difference between the normal diet and the high-salt diet doesn’t matter, as long as the difference is there. Whether the rats were given 0.00001mg or 1000kg of more salt, we aren’t looking for the degree of change but whether the change is actually there or not.

B) The conclusion is only explicit about the relationship between high-salt diet and high blood pressure. Any consequences caused by the high blood pressure is therefore OOS

C) We only care the colony in the experiment, and it’s not important what other naturally occurring colonies’ diet consists of. I think this could’ve been a much better answer if it asked, "What percentage of the observed colony naturally feed on high-salt diets?"

E) Other species are OOS.
 
asafezrati
Thanks Received: 6
Atticus Finch
Atticus Finch
 
Posts: 116
Joined: December 07th, 2014
 
 
 

Re: Q3 - A recent study involved

by asafezrati Wed Apr 29, 2015 2:12 pm

mattsherman Wrote:I see your point, but I think the issue you see isn't really there. This is a study of a colony of rats. I think it's fair to say that we know how many of the rats are in the study, and thus the colony.

Keep that sharp eye peeled for later in the section. Your pulling out too critical of an eye too early in the section! Good luck.

I can subscribe to answer choice D since the question stem said "most", and because the other answer choices don't help us evaluate.

But I do think that the shift between % and numbers is a big one, and can't see how we can safely say that we know the numerical details. We are judging the argument and the information included in it (which clearly says nothing about actual quantity). We don't know anything else besides percentages.

Can someone give another opinion on this shift?
 
christine.defenbaugh
Thanks Received: 585
Atticus Finch
Atticus Finch
 
Posts: 536
Joined: May 17th, 2013
 
 
 

Re: Q3 - A recent study involved

by christine.defenbaugh Wed May 06, 2015 5:10 pm

Terribly interesting question, asafezrati!

First, I want to caution you never to let your understanding of an answer off the hook by resorting to 'well, the question just said 'most''. Since (D) has to clearly help us to evaluate, we've got to keep honing our understanding of the issue until we see why that's the case.

Evaluate questions are a bit unusual on the LSAT. I like to think of them as strengthen/weaken questions. For something to be useful to evaluate, I need at least one answer out there to weaken the argument, and at least one answer to strengthen the argument. The easiest way to find these it to consider the most extreme possible answers to the question: your best and worst case scenarios.

Let's break down the argument first:
    PREMISE:
    1) experiment: rats + high-salt diet
    2) after a few months: 25% normal BP, 70% HBP, 5% extremely HBP

    CONCLUSION: high-salt diets are linked to HBP in rats


Once we break it down, we should see that we have only been given the 'after' snapshot. Without the 'before' snapshot, there's no way to know whether 'after' scenario was actually an increase in HBP, a decrease in HBP, or whether everything stayed pretty stable. The argument assumes there was an increase in the HBP stats! It would be REALLY great to know the percentage of the rats that had HBP beforehand, so that we could compare that number to 70 percent - if we had that, we'd totally be able to lock this argument down as either valid or invalid.

(D) should be immediately tempting because it raises the idea of the 'before' snapshot. But we need to investigate whether this question actually has both strengthening and weakening potential answers. The fact that it doesn't raise percentage is a valid concern, but may not be fatal.

What's the worst case scenario answer to the question? Zero! That's a totally valid answer the question "how many?" If the answer is that ZERO rats had HBP before the study began, then that would strengthen this argument tons!

Okay, so what about the best case? The opposite extreme is "all of them". While that's not a numeral, it is a reasonable answer to the question "how many?".
    Me: How many of the chocolate chip cookies did you eat??
    My fiance: Umm....all of them?
    Me: :o

If ALL of the rats had HBP beforehand, then this high-salt diet actually decreased their BP, and the argument is disastrously undermined.

There are lots of potential answers to the question that would not help us evaluate this argument (such as 57 or 577 :mrgreen: ), but we don't need a question that would guarantee an evaluation with every possible answer. All we need is a question that has the potential to both strengthen and weaken. If our best and worst case scenarios can give us that, that's good enough to 'help' to evaluate the argument.

A question that asked for the percent might be more useful, in that more of the potential answers would clearly strengthen or weaken, but fortunately, we never have to weigh which question helps more. The LSAT will never give us two responses that both do the job, but one does it 'better'.

Let's take a spin through the UNhelpful ones:

    (A) We know that's it's 'high-salt' from the premises. That means it's more salt than the regular diet. It doesn't matter HOW MUCH more salt. Even a teeny bit more salt could support this conclusion, if there's a real increase in HBP.
    (B) Adverse health effects are out of scope. This conclusion is only about a potential link between high-salt and HBP. How that HBP messes you up later is a conversation for later!
    (C) This is about other rat colonies. It wouldn't tell me anything about what's going on with this colony in this experiment.
    (E) This is about other rodents and other experiments - I want to know about this experiment and these rodents (specifically, these rats)!


Please let me know if this helps clear up a few things, particularly the percents vs numbers issue!